Are fly wheels a valid means of energy storage for powering a large space station?












9












$begingroup$


This is something that was just offhandedly mentioned in a Sci-Fi book I read once and as I am now writing one myself I thought I might use it.



The scenario is that I have a ship approaching a warp gate and am describing the appearance and mechanics of the gate. The idea is that warp drive research on Earth ended in a dead end because we were never able to effectively produce enough energy to power a warp field larger than the ship and generator itself.



Jump to 400 hundred years in the future and humanity has colonized a few other star systems by means of sub-light, generation ships. They then develop a functioning warp bridge, rather than a ship drive, by building massive warp field generators on each end.



The generator stations, basically warp gates for simplicity, generate the necessary energy over several days, and then expend it all in a single day or a few hours (undecided) to power the warp field.



Would massive flywheels be a valid form of energy storage? If not, what other ideas of energy storage might be more reasonable?



I would like to clarify that this is the heaviest handwavium in my entire story. Besides the completion of warp theory, functioning cryogenics, fusion drive technology, and some advances in material science, the story is supposed to be as reasonably realistic as I can get. Real physics. No artificial gravity. Space travel takes days and weeks between planets in-system. The warp gates are posted at the far edge of a solar system, allowing instantaneous travel between systems, but once exiting the gate traveling in-system is much slower again.



Addendum: I should have included that I am imagining two flywheels in a pair on opposite sides of the station to counteract one another, as many of you have mentioned that would be necessary.










share|improve this question











$endgroup$








  • 1




    $begingroup$
    Flywheels are in fact used for various applications that need high, short-duration current without affecting the local grid: en.wikipedia.org/wiki/Flywheel_energy_storage#Test_laboratories So having a large one in your warp station is simple extrapolation.
    $endgroup$
    – jamesqf
    Dec 21 '18 at 18:32










  • $begingroup$
    Here's another article detailing the use of Flywheels and Batteries for high impact energy usage. ktoo.org/2017/09/15/… I'm not smart enough to answer whether or not that can be extrapolated.
    $endgroup$
    – GuidingOlive
    Dec 21 '18 at 19:59






  • 2




    $begingroup$
    What's so wrong with supercapacitors that makes you want to store energy in a massive rotating mechanical device, with all its nasty side effects? (It acts as a massive giroscope; rotational momentum is a conserved quantity, so when you spin the flywheel up or down something else must spin in the opposite direction; it's heaaaavy...)
    $endgroup$
    – AlexP
    Dec 21 '18 at 21:21








  • 1




    $begingroup$
    I would like to note that one of the important things to have in mind while writing a sci-fi story is (in my opinion) to try to make things as convenient as possible. Many of stories that appear "hilarious" today come from the author's idea to take what we have today and add something to make it better. Cars? Same ones, but they can fly. Robots? Still as awkward, but they can talk. Energy source? Flywheels, but they are bigger... Addressing a core issue in not realistic (for the times), but much more convenient way is imho the key.
    $endgroup$
    – MatthewRock
    Dec 21 '18 at 23:49






  • 1




    $begingroup$
    Remember to use your flywheels in pairs (either counter-rotating or on opposite sides of the station) to balance the forces, lest your warp facility start drunkenly tumbling.
    $endgroup$
    – user535733
    Dec 22 '18 at 3:29
















9












$begingroup$


This is something that was just offhandedly mentioned in a Sci-Fi book I read once and as I am now writing one myself I thought I might use it.



The scenario is that I have a ship approaching a warp gate and am describing the appearance and mechanics of the gate. The idea is that warp drive research on Earth ended in a dead end because we were never able to effectively produce enough energy to power a warp field larger than the ship and generator itself.



Jump to 400 hundred years in the future and humanity has colonized a few other star systems by means of sub-light, generation ships. They then develop a functioning warp bridge, rather than a ship drive, by building massive warp field generators on each end.



The generator stations, basically warp gates for simplicity, generate the necessary energy over several days, and then expend it all in a single day or a few hours (undecided) to power the warp field.



Would massive flywheels be a valid form of energy storage? If not, what other ideas of energy storage might be more reasonable?



I would like to clarify that this is the heaviest handwavium in my entire story. Besides the completion of warp theory, functioning cryogenics, fusion drive technology, and some advances in material science, the story is supposed to be as reasonably realistic as I can get. Real physics. No artificial gravity. Space travel takes days and weeks between planets in-system. The warp gates are posted at the far edge of a solar system, allowing instantaneous travel between systems, but once exiting the gate traveling in-system is much slower again.



Addendum: I should have included that I am imagining two flywheels in a pair on opposite sides of the station to counteract one another, as many of you have mentioned that would be necessary.










share|improve this question











$endgroup$








  • 1




    $begingroup$
    Flywheels are in fact used for various applications that need high, short-duration current without affecting the local grid: en.wikipedia.org/wiki/Flywheel_energy_storage#Test_laboratories So having a large one in your warp station is simple extrapolation.
    $endgroup$
    – jamesqf
    Dec 21 '18 at 18:32










  • $begingroup$
    Here's another article detailing the use of Flywheels and Batteries for high impact energy usage. ktoo.org/2017/09/15/… I'm not smart enough to answer whether or not that can be extrapolated.
    $endgroup$
    – GuidingOlive
    Dec 21 '18 at 19:59






  • 2




    $begingroup$
    What's so wrong with supercapacitors that makes you want to store energy in a massive rotating mechanical device, with all its nasty side effects? (It acts as a massive giroscope; rotational momentum is a conserved quantity, so when you spin the flywheel up or down something else must spin in the opposite direction; it's heaaaavy...)
    $endgroup$
    – AlexP
    Dec 21 '18 at 21:21








  • 1




    $begingroup$
    I would like to note that one of the important things to have in mind while writing a sci-fi story is (in my opinion) to try to make things as convenient as possible. Many of stories that appear "hilarious" today come from the author's idea to take what we have today and add something to make it better. Cars? Same ones, but they can fly. Robots? Still as awkward, but they can talk. Energy source? Flywheels, but they are bigger... Addressing a core issue in not realistic (for the times), but much more convenient way is imho the key.
    $endgroup$
    – MatthewRock
    Dec 21 '18 at 23:49






  • 1




    $begingroup$
    Remember to use your flywheels in pairs (either counter-rotating or on opposite sides of the station) to balance the forces, lest your warp facility start drunkenly tumbling.
    $endgroup$
    – user535733
    Dec 22 '18 at 3:29














9












9








9


4



$begingroup$


This is something that was just offhandedly mentioned in a Sci-Fi book I read once and as I am now writing one myself I thought I might use it.



The scenario is that I have a ship approaching a warp gate and am describing the appearance and mechanics of the gate. The idea is that warp drive research on Earth ended in a dead end because we were never able to effectively produce enough energy to power a warp field larger than the ship and generator itself.



Jump to 400 hundred years in the future and humanity has colonized a few other star systems by means of sub-light, generation ships. They then develop a functioning warp bridge, rather than a ship drive, by building massive warp field generators on each end.



The generator stations, basically warp gates for simplicity, generate the necessary energy over several days, and then expend it all in a single day or a few hours (undecided) to power the warp field.



Would massive flywheels be a valid form of energy storage? If not, what other ideas of energy storage might be more reasonable?



I would like to clarify that this is the heaviest handwavium in my entire story. Besides the completion of warp theory, functioning cryogenics, fusion drive technology, and some advances in material science, the story is supposed to be as reasonably realistic as I can get. Real physics. No artificial gravity. Space travel takes days and weeks between planets in-system. The warp gates are posted at the far edge of a solar system, allowing instantaneous travel between systems, but once exiting the gate traveling in-system is much slower again.



Addendum: I should have included that I am imagining two flywheels in a pair on opposite sides of the station to counteract one another, as many of you have mentioned that would be necessary.










share|improve this question











$endgroup$




This is something that was just offhandedly mentioned in a Sci-Fi book I read once and as I am now writing one myself I thought I might use it.



The scenario is that I have a ship approaching a warp gate and am describing the appearance and mechanics of the gate. The idea is that warp drive research on Earth ended in a dead end because we were never able to effectively produce enough energy to power a warp field larger than the ship and generator itself.



Jump to 400 hundred years in the future and humanity has colonized a few other star systems by means of sub-light, generation ships. They then develop a functioning warp bridge, rather than a ship drive, by building massive warp field generators on each end.



The generator stations, basically warp gates for simplicity, generate the necessary energy over several days, and then expend it all in a single day or a few hours (undecided) to power the warp field.



Would massive flywheels be a valid form of energy storage? If not, what other ideas of energy storage might be more reasonable?



I would like to clarify that this is the heaviest handwavium in my entire story. Besides the completion of warp theory, functioning cryogenics, fusion drive technology, and some advances in material science, the story is supposed to be as reasonably realistic as I can get. Real physics. No artificial gravity. Space travel takes days and weeks between planets in-system. The warp gates are posted at the far edge of a solar system, allowing instantaneous travel between systems, but once exiting the gate traveling in-system is much slower again.



Addendum: I should have included that I am imagining two flywheels in a pair on opposite sides of the station to counteract one another, as many of you have mentioned that would be necessary.







physics space-travel space-constructs engineering






share|improve this question















share|improve this question













share|improve this question




share|improve this question








edited Dec 22 '18 at 5:01







TitaniumTurtle

















asked Dec 21 '18 at 17:29









TitaniumTurtleTitaniumTurtle

1,729529




1,729529








  • 1




    $begingroup$
    Flywheels are in fact used for various applications that need high, short-duration current without affecting the local grid: en.wikipedia.org/wiki/Flywheel_energy_storage#Test_laboratories So having a large one in your warp station is simple extrapolation.
    $endgroup$
    – jamesqf
    Dec 21 '18 at 18:32










  • $begingroup$
    Here's another article detailing the use of Flywheels and Batteries for high impact energy usage. ktoo.org/2017/09/15/… I'm not smart enough to answer whether or not that can be extrapolated.
    $endgroup$
    – GuidingOlive
    Dec 21 '18 at 19:59






  • 2




    $begingroup$
    What's so wrong with supercapacitors that makes you want to store energy in a massive rotating mechanical device, with all its nasty side effects? (It acts as a massive giroscope; rotational momentum is a conserved quantity, so when you spin the flywheel up or down something else must spin in the opposite direction; it's heaaaavy...)
    $endgroup$
    – AlexP
    Dec 21 '18 at 21:21








  • 1




    $begingroup$
    I would like to note that one of the important things to have in mind while writing a sci-fi story is (in my opinion) to try to make things as convenient as possible. Many of stories that appear "hilarious" today come from the author's idea to take what we have today and add something to make it better. Cars? Same ones, but they can fly. Robots? Still as awkward, but they can talk. Energy source? Flywheels, but they are bigger... Addressing a core issue in not realistic (for the times), but much more convenient way is imho the key.
    $endgroup$
    – MatthewRock
    Dec 21 '18 at 23:49






  • 1




    $begingroup$
    Remember to use your flywheels in pairs (either counter-rotating or on opposite sides of the station) to balance the forces, lest your warp facility start drunkenly tumbling.
    $endgroup$
    – user535733
    Dec 22 '18 at 3:29














  • 1




    $begingroup$
    Flywheels are in fact used for various applications that need high, short-duration current without affecting the local grid: en.wikipedia.org/wiki/Flywheel_energy_storage#Test_laboratories So having a large one in your warp station is simple extrapolation.
    $endgroup$
    – jamesqf
    Dec 21 '18 at 18:32










  • $begingroup$
    Here's another article detailing the use of Flywheels and Batteries for high impact energy usage. ktoo.org/2017/09/15/… I'm not smart enough to answer whether or not that can be extrapolated.
    $endgroup$
    – GuidingOlive
    Dec 21 '18 at 19:59






  • 2




    $begingroup$
    What's so wrong with supercapacitors that makes you want to store energy in a massive rotating mechanical device, with all its nasty side effects? (It acts as a massive giroscope; rotational momentum is a conserved quantity, so when you spin the flywheel up or down something else must spin in the opposite direction; it's heaaaavy...)
    $endgroup$
    – AlexP
    Dec 21 '18 at 21:21








  • 1




    $begingroup$
    I would like to note that one of the important things to have in mind while writing a sci-fi story is (in my opinion) to try to make things as convenient as possible. Many of stories that appear "hilarious" today come from the author's idea to take what we have today and add something to make it better. Cars? Same ones, but they can fly. Robots? Still as awkward, but they can talk. Energy source? Flywheels, but they are bigger... Addressing a core issue in not realistic (for the times), but much more convenient way is imho the key.
    $endgroup$
    – MatthewRock
    Dec 21 '18 at 23:49






  • 1




    $begingroup$
    Remember to use your flywheels in pairs (either counter-rotating or on opposite sides of the station) to balance the forces, lest your warp facility start drunkenly tumbling.
    $endgroup$
    – user535733
    Dec 22 '18 at 3:29








1




1




$begingroup$
Flywheels are in fact used for various applications that need high, short-duration current without affecting the local grid: en.wikipedia.org/wiki/Flywheel_energy_storage#Test_laboratories So having a large one in your warp station is simple extrapolation.
$endgroup$
– jamesqf
Dec 21 '18 at 18:32




$begingroup$
Flywheels are in fact used for various applications that need high, short-duration current without affecting the local grid: en.wikipedia.org/wiki/Flywheel_energy_storage#Test_laboratories So having a large one in your warp station is simple extrapolation.
$endgroup$
– jamesqf
Dec 21 '18 at 18:32












$begingroup$
Here's another article detailing the use of Flywheels and Batteries for high impact energy usage. ktoo.org/2017/09/15/… I'm not smart enough to answer whether or not that can be extrapolated.
$endgroup$
– GuidingOlive
Dec 21 '18 at 19:59




$begingroup$
Here's another article detailing the use of Flywheels and Batteries for high impact energy usage. ktoo.org/2017/09/15/… I'm not smart enough to answer whether or not that can be extrapolated.
$endgroup$
– GuidingOlive
Dec 21 '18 at 19:59




2




2




$begingroup$
What's so wrong with supercapacitors that makes you want to store energy in a massive rotating mechanical device, with all its nasty side effects? (It acts as a massive giroscope; rotational momentum is a conserved quantity, so when you spin the flywheel up or down something else must spin in the opposite direction; it's heaaaavy...)
$endgroup$
– AlexP
Dec 21 '18 at 21:21






$begingroup$
What's so wrong with supercapacitors that makes you want to store energy in a massive rotating mechanical device, with all its nasty side effects? (It acts as a massive giroscope; rotational momentum is a conserved quantity, so when you spin the flywheel up or down something else must spin in the opposite direction; it's heaaaavy...)
$endgroup$
– AlexP
Dec 21 '18 at 21:21






1




1




$begingroup$
I would like to note that one of the important things to have in mind while writing a sci-fi story is (in my opinion) to try to make things as convenient as possible. Many of stories that appear "hilarious" today come from the author's idea to take what we have today and add something to make it better. Cars? Same ones, but they can fly. Robots? Still as awkward, but they can talk. Energy source? Flywheels, but they are bigger... Addressing a core issue in not realistic (for the times), but much more convenient way is imho the key.
$endgroup$
– MatthewRock
Dec 21 '18 at 23:49




$begingroup$
I would like to note that one of the important things to have in mind while writing a sci-fi story is (in my opinion) to try to make things as convenient as possible. Many of stories that appear "hilarious" today come from the author's idea to take what we have today and add something to make it better. Cars? Same ones, but they can fly. Robots? Still as awkward, but they can talk. Energy source? Flywheels, but they are bigger... Addressing a core issue in not realistic (for the times), but much more convenient way is imho the key.
$endgroup$
– MatthewRock
Dec 21 '18 at 23:49




1




1




$begingroup$
Remember to use your flywheels in pairs (either counter-rotating or on opposite sides of the station) to balance the forces, lest your warp facility start drunkenly tumbling.
$endgroup$
– user535733
Dec 22 '18 at 3:29




$begingroup$
Remember to use your flywheels in pairs (either counter-rotating or on opposite sides of the station) to balance the forces, lest your warp facility start drunkenly tumbling.
$endgroup$
– user535733
Dec 22 '18 at 3:29










6 Answers
6






active

oldest

votes


















16












$begingroup$

The answer is almost certainly no, but here's your state of the art:



Superconducting Flywheel



This flywheel built around superconducting magnetic bearings is about 2m wide and 4 tons. It holds 100kW-h and can output it at 300kW, which means it can dump all of its energy in somewhere around 20-30 minutes.



You mentioned that warp drive tech failed to be developed on Earth because they couldn't get the power. Well that means we can draw meaningful comparisons to Earth power plants, like Kashiwazaki-Kariwa Nuclear Power Station in Japan. That mighty force can generate 8,212,000kW of power. This means you need 27,000 of those superconducting flywheels to match her output.



One of the major limits to effectiveness of modern flywheels is tensile strength. As you store more power, you put more stress on the material. At some point, the flywheel tears itself apart. (By the way, you want to make sure these flywheels are well insulated from one another... a chain reaction of exploding flywheels is a horrible sight to see. Here's what a small one can do).



Also consider that we can generate higher power discharges. Power plants are designed for generating continous power. If you just need a discrete shock, you may want to look at the Z-pinch machines like the Z-Machine at Sandia labs



Z machine



Besides looking freaking awesome in pictures, the Z-machine is capable of generating pulses as strong as 350 TW (That's 350,000,000,000 kW, if we keep the kW units we used for flywheels), and there are plans on the drawing board for a new device that can hit 1PW (1,000,000,000,000 kW).






share|improve this answer











$endgroup$





















    6












    $begingroup$

    Yes, but...



    Flywheels are being used to store energy today, only not in any great amounts, and not as means of transporting energy from one place to another.



    enter image description here



    That big circle is a flywheel, acting as a sort of energy buffer between the strokes of the single cylinder of the locomotive.



    In general, storing energy as momentum is a bad idea. You cannot store a lot of it before the flywheel is spinning so fast it tears itself apart, and unless you solve friction issues, you will quickly lose it as heat.



    Also you get another problem: gyroscopic forces. Conservation of moment of inertia means that means you must mount these flywheels on some kind of gimbal, or you will never be able to turn your ship.



    Finally, I must point out the inherent danger of having a huge rotating object inside your ship. Unlike chemical or nuclear fuel, you cannot just dump the energy and be rid of it. Also it means that your transport medium is extremely volatile. One thing that goes wrong with this... and you are in for a very bad time.






    share|improve this answer









    $endgroup$









    • 1




      $begingroup$
      You are right that stored power in flywheels is dangerous, but large amounts stored power is always potentially dangerous if that power can ever be released in an uncontrolled manner.
      $endgroup$
      – Gary Walker
      Dec 21 '18 at 20:01










    • $begingroup$
      Given the vacuum in space, I think you'd be better storing energy as heat instead of a rotating body.
      $endgroup$
      – Jiminion
      Dec 21 '18 at 21:45










    • $begingroup$
      @GaryWalker Which is why we will never see a supercapacitor car. The energy is a typical automotive gas tank is similar to the energy in a normal 2000# military bomb. (Remember, a good portion of that weight is case, not boom.) Imagine if you had a traffic jam and something shorted one of them out.
      $endgroup$
      – Loren Pechtel
      Dec 21 '18 at 21:47










    • $begingroup$
      Wouldn't the flywheels need to be used as contrarotating pairs to avoid changing the spin rate of the space station? (Space station Earth is large enough that they can be used individually.)
      $endgroup$
      – Andrew Morton
      Dec 21 '18 at 21:58










    • $begingroup$
      Actually, flywheels work quite well as energy storage/transport, for instance the Gyrobus en.wikipedia.org/wiki/Gyrobus or the use of flywheels for energy storage in race cars popularmechanics.com/cars/hybrid-electric/a6266/… And of course every IC-engined car uses a flywheel as an energy buffer.
      $endgroup$
      – jamesqf
      Dec 23 '18 at 20:31



















    6












    $begingroup$

    It is possible, but maybe not the best idea



    Modern flywheels can accumulate a good amount of energy Energy densities, and they are relatively safe (meaning, they won't explosively disintegrate like solid flywheels can do), but they trail other means of energy storage in terms of energy density. Flywheel's specific energy, in MJ/kg, goes to 0.5, which is on par with regular Li-ion batteries, worse than Lithium metal batteries (1.8), and way, way worse than Hydrogen fuel cells (142). Actually, Wiki's figure for hydrogen assumes Earth conditions, where oxygen is plentiful. In spaceships, oxygen storage is also required. This would make the whole system several times less efficient per unit of mass, but still much better than a flywheel.



    Flywheels are good when energy comes and goes in mechanical form. If you need the energy in a different form (to power the warp field) there will be extra overhead that would further negate their advantages.



    But for creating a "streampunk universe", why not, go with flywheels!






    share|improve this answer











    $endgroup$





















      4












      $begingroup$

      Flywheels are not ideal for your application in the 25th century.



      Due to materials advances, flywheels would have a larger capacity than today, but there are 2 reasonable tech advances that would likely be much nicer.



      Super-capacitors have already been proposed that have energy densities equal to fuel cells. They also have power densities that are orders of magnitudes better. It is hard to predict just how powerful these could become with an additional 400 years of advances.



      Room temperature superconductors may lead to high capacity storage as well.



      Both of these technologies are inherently electrical instead of mechanical, and potentially more usable for directly powering a warp field.



      You specify powering a warp gate for hours or more, these technologies would be suitable for this, or for powering warp gates for minutes or even seconds.



      Flywheels are rarely used as a power source for hours, they are more commonly used as a power source for minutes or seconds until a secondary power source can respond (such as a diesel generator). This is because the storage densities of flywheels is relatively low.



      But spinning metal is very good at averaging power fluctuations. In fact, spinning metal in the generators themselves is a large component in power averaging the current electric grid.



      If you want another solution, consider the possibility of using hafnium, specifically 178m2Hf. You need technical advances to figure out how to release the stored energy when desired, but the storage potential is huge. This adds complication in radio-activity and how to convert the gamma energy-release into something for powering your warp-generators, but this could be woven into an interesting plot-line.






      share|improve this answer











      $endgroup$





















        3












        $begingroup$

        Your very awesome flywheel is made of neutronium.



        binary neutron stars



        https://newscenter.lbl.gov/2017/10/16/scientists-decode-signature-neutron-star-merger/



        A flywheel will tear itself apart if it rotates too fast. Unless the forces holding the flywheel together are super strong - as is the case for neutron stars. Neutron stars will teach you about fast spinning.




        As the star's core collapses, its rotation rate increases as a result
        of conservation of angular momentum, hence newly formed neutron stars
        rotate at up to several hundred times per second.




        The neutronium comprising neutron stars is so dense that it will stay together even at relativistic rotation speeds. A 10 km neutron star with the mass of the Sun might be a little unwieldy for your application. I wondered if a smaller chunk of neutronium could still hold together. According to the physics stack, maybe.



        https://physics.stackexchange.com/questions/143166/what-is-the-theoretical-lower-mass-limit-for-a-gravitationally-stable-neutron-st




        Theoretically a stable neutron star could exist with a much lower
        mass, if one could work out a way of forming it (perhaps in a close
        binary neutron star where one component loses mass to the other prior
        to a merger?).




        It goes on with some hard core physics, including why a smaller amount of neutronium might make a larger object that degenerates into more commonplace matter.



        Or maybe you would site your stargates at naturally occurring binary neutron stars, as depicted above. In any case - your flywheel is a spinning chunk of neutronium. Or two spinning chunks, to form the halves of your gate as noted in the OP. Given their mass they will be orbiting around each other. Given their proximity they will be moving very, very fast and they will look very very cool. You can approach perpendicular to their plane of motion and pass between them. You had better stay in the exact center. Stars get grabby.



        How does one get energy from a fast rotating neutron star? Maybe induce it to shoot out a jet of energy that you harness for your purposes.



        https://www.sciencedaily.com/releases/2018/09/180926140829.htm




        Another idea, they said, is that the jets may be powered by the
        neutron star's rotation, instead of being launched by magnetic field
        lines in the inner accretion disk.



        "Interestingly, the rotation-powered idea predicts that the jet will
        be significantly weaker from more slowly rotating neutron stars, which
        is exactly what we see in Sw J0243," Nathalie Degenaar, also of the
        University of Amsterdam, said.







        share|improve this answer











        $endgroup$





















          0












          $begingroup$

          If you need all your energy to be released in one go (is in the case of generator stations) you have significantly fewer options than normal. Any thermal method of energy generation will likely be too hot for most materials and batteries are simply too slow to release your energy. This leaves you with just capacitors, flywheels and maybe handwavium if you've got it. Though in all honesty this explanation requires the bulk your energy to be released very quickly for batteries to be non-competitive.



          (in space) Flywheels are very very cheap. This is because they are incredibly simple. if you find a significantly massive asteroid (of which there will probably be millions in any given solar system) you can simply smelt it into steel (quite cheap given the awsome source of energy that is fusion) and then BAMN flywheel, much more processing would be required for a given energy of capacitor bank.) The flywheel in question would be massive though.



          With good barings your flywheel will basically lose no energy to the outside would



          Lets say you have a 100 MW reactor given a energy density of 32 Watt-hours per kilogram storing energy over a week then that's going to be 500000 tons of steel. That's about 70 effiel towers. This is definitely doable just make sure to have your flywheels by BIG.






          share|improve this answer









          $endgroup$













            Your Answer





            StackExchange.ifUsing("editor", function () {
            return StackExchange.using("mathjaxEditing", function () {
            StackExchange.MarkdownEditor.creationCallbacks.add(function (editor, postfix) {
            StackExchange.mathjaxEditing.prepareWmdForMathJax(editor, postfix, [["$", "$"], ["\\(","\\)"]]);
            });
            });
            }, "mathjax-editing");

            StackExchange.ready(function() {
            var channelOptions = {
            tags: "".split(" "),
            id: "579"
            };
            initTagRenderer("".split(" "), "".split(" "), channelOptions);

            StackExchange.using("externalEditor", function() {
            // Have to fire editor after snippets, if snippets enabled
            if (StackExchange.settings.snippets.snippetsEnabled) {
            StackExchange.using("snippets", function() {
            createEditor();
            });
            }
            else {
            createEditor();
            }
            });

            function createEditor() {
            StackExchange.prepareEditor({
            heartbeatType: 'answer',
            autoActivateHeartbeat: false,
            convertImagesToLinks: false,
            noModals: true,
            showLowRepImageUploadWarning: true,
            reputationToPostImages: null,
            bindNavPrevention: true,
            postfix: "",
            imageUploader: {
            brandingHtml: "Powered by u003ca class="icon-imgur-white" href="https://imgur.com/"u003eu003c/au003e",
            contentPolicyHtml: "User contributions licensed under u003ca href="https://creativecommons.org/licenses/by-sa/3.0/"u003ecc by-sa 3.0 with attribution requiredu003c/au003e u003ca href="https://stackoverflow.com/legal/content-policy"u003e(content policy)u003c/au003e",
            allowUrls: true
            },
            noCode: true, onDemand: true,
            discardSelector: ".discard-answer"
            ,immediatelyShowMarkdownHelp:true
            });


            }
            });














            draft saved

            draft discarded


















            StackExchange.ready(
            function () {
            StackExchange.openid.initPostLogin('.new-post-login', 'https%3a%2f%2fworldbuilding.stackexchange.com%2fquestions%2f134515%2fare-fly-wheels-a-valid-means-of-energy-storage-for-powering-a-large-space-statio%23new-answer', 'question_page');
            }
            );

            Post as a guest















            Required, but never shown

























            6 Answers
            6






            active

            oldest

            votes








            6 Answers
            6






            active

            oldest

            votes









            active

            oldest

            votes






            active

            oldest

            votes









            16












            $begingroup$

            The answer is almost certainly no, but here's your state of the art:



            Superconducting Flywheel



            This flywheel built around superconducting magnetic bearings is about 2m wide and 4 tons. It holds 100kW-h and can output it at 300kW, which means it can dump all of its energy in somewhere around 20-30 minutes.



            You mentioned that warp drive tech failed to be developed on Earth because they couldn't get the power. Well that means we can draw meaningful comparisons to Earth power plants, like Kashiwazaki-Kariwa Nuclear Power Station in Japan. That mighty force can generate 8,212,000kW of power. This means you need 27,000 of those superconducting flywheels to match her output.



            One of the major limits to effectiveness of modern flywheels is tensile strength. As you store more power, you put more stress on the material. At some point, the flywheel tears itself apart. (By the way, you want to make sure these flywheels are well insulated from one another... a chain reaction of exploding flywheels is a horrible sight to see. Here's what a small one can do).



            Also consider that we can generate higher power discharges. Power plants are designed for generating continous power. If you just need a discrete shock, you may want to look at the Z-pinch machines like the Z-Machine at Sandia labs



            Z machine



            Besides looking freaking awesome in pictures, the Z-machine is capable of generating pulses as strong as 350 TW (That's 350,000,000,000 kW, if we keep the kW units we used for flywheels), and there are plans on the drawing board for a new device that can hit 1PW (1,000,000,000,000 kW).






            share|improve this answer











            $endgroup$


















              16












              $begingroup$

              The answer is almost certainly no, but here's your state of the art:



              Superconducting Flywheel



              This flywheel built around superconducting magnetic bearings is about 2m wide and 4 tons. It holds 100kW-h and can output it at 300kW, which means it can dump all of its energy in somewhere around 20-30 minutes.



              You mentioned that warp drive tech failed to be developed on Earth because they couldn't get the power. Well that means we can draw meaningful comparisons to Earth power plants, like Kashiwazaki-Kariwa Nuclear Power Station in Japan. That mighty force can generate 8,212,000kW of power. This means you need 27,000 of those superconducting flywheels to match her output.



              One of the major limits to effectiveness of modern flywheels is tensile strength. As you store more power, you put more stress on the material. At some point, the flywheel tears itself apart. (By the way, you want to make sure these flywheels are well insulated from one another... a chain reaction of exploding flywheels is a horrible sight to see. Here's what a small one can do).



              Also consider that we can generate higher power discharges. Power plants are designed for generating continous power. If you just need a discrete shock, you may want to look at the Z-pinch machines like the Z-Machine at Sandia labs



              Z machine



              Besides looking freaking awesome in pictures, the Z-machine is capable of generating pulses as strong as 350 TW (That's 350,000,000,000 kW, if we keep the kW units we used for flywheels), and there are plans on the drawing board for a new device that can hit 1PW (1,000,000,000,000 kW).






              share|improve this answer











              $endgroup$
















                16












                16








                16





                $begingroup$

                The answer is almost certainly no, but here's your state of the art:



                Superconducting Flywheel



                This flywheel built around superconducting magnetic bearings is about 2m wide and 4 tons. It holds 100kW-h and can output it at 300kW, which means it can dump all of its energy in somewhere around 20-30 minutes.



                You mentioned that warp drive tech failed to be developed on Earth because they couldn't get the power. Well that means we can draw meaningful comparisons to Earth power plants, like Kashiwazaki-Kariwa Nuclear Power Station in Japan. That mighty force can generate 8,212,000kW of power. This means you need 27,000 of those superconducting flywheels to match her output.



                One of the major limits to effectiveness of modern flywheels is tensile strength. As you store more power, you put more stress on the material. At some point, the flywheel tears itself apart. (By the way, you want to make sure these flywheels are well insulated from one another... a chain reaction of exploding flywheels is a horrible sight to see. Here's what a small one can do).



                Also consider that we can generate higher power discharges. Power plants are designed for generating continous power. If you just need a discrete shock, you may want to look at the Z-pinch machines like the Z-Machine at Sandia labs



                Z machine



                Besides looking freaking awesome in pictures, the Z-machine is capable of generating pulses as strong as 350 TW (That's 350,000,000,000 kW, if we keep the kW units we used for flywheels), and there are plans on the drawing board for a new device that can hit 1PW (1,000,000,000,000 kW).






                share|improve this answer











                $endgroup$



                The answer is almost certainly no, but here's your state of the art:



                Superconducting Flywheel



                This flywheel built around superconducting magnetic bearings is about 2m wide and 4 tons. It holds 100kW-h and can output it at 300kW, which means it can dump all of its energy in somewhere around 20-30 minutes.



                You mentioned that warp drive tech failed to be developed on Earth because they couldn't get the power. Well that means we can draw meaningful comparisons to Earth power plants, like Kashiwazaki-Kariwa Nuclear Power Station in Japan. That mighty force can generate 8,212,000kW of power. This means you need 27,000 of those superconducting flywheels to match her output.



                One of the major limits to effectiveness of modern flywheels is tensile strength. As you store more power, you put more stress on the material. At some point, the flywheel tears itself apart. (By the way, you want to make sure these flywheels are well insulated from one another... a chain reaction of exploding flywheels is a horrible sight to see. Here's what a small one can do).



                Also consider that we can generate higher power discharges. Power plants are designed for generating continous power. If you just need a discrete shock, you may want to look at the Z-pinch machines like the Z-Machine at Sandia labs



                Z machine



                Besides looking freaking awesome in pictures, the Z-machine is capable of generating pulses as strong as 350 TW (That's 350,000,000,000 kW, if we keep the kW units we used for flywheels), and there are plans on the drawing board for a new device that can hit 1PW (1,000,000,000,000 kW).







                share|improve this answer














                share|improve this answer



                share|improve this answer








                edited Dec 21 '18 at 20:44

























                answered Dec 21 '18 at 20:38









                Cort AmmonCort Ammon

                109k17188385




                109k17188385























                    6












                    $begingroup$

                    Yes, but...



                    Flywheels are being used to store energy today, only not in any great amounts, and not as means of transporting energy from one place to another.



                    enter image description here



                    That big circle is a flywheel, acting as a sort of energy buffer between the strokes of the single cylinder of the locomotive.



                    In general, storing energy as momentum is a bad idea. You cannot store a lot of it before the flywheel is spinning so fast it tears itself apart, and unless you solve friction issues, you will quickly lose it as heat.



                    Also you get another problem: gyroscopic forces. Conservation of moment of inertia means that means you must mount these flywheels on some kind of gimbal, or you will never be able to turn your ship.



                    Finally, I must point out the inherent danger of having a huge rotating object inside your ship. Unlike chemical or nuclear fuel, you cannot just dump the energy and be rid of it. Also it means that your transport medium is extremely volatile. One thing that goes wrong with this... and you are in for a very bad time.






                    share|improve this answer









                    $endgroup$









                    • 1




                      $begingroup$
                      You are right that stored power in flywheels is dangerous, but large amounts stored power is always potentially dangerous if that power can ever be released in an uncontrolled manner.
                      $endgroup$
                      – Gary Walker
                      Dec 21 '18 at 20:01










                    • $begingroup$
                      Given the vacuum in space, I think you'd be better storing energy as heat instead of a rotating body.
                      $endgroup$
                      – Jiminion
                      Dec 21 '18 at 21:45










                    • $begingroup$
                      @GaryWalker Which is why we will never see a supercapacitor car. The energy is a typical automotive gas tank is similar to the energy in a normal 2000# military bomb. (Remember, a good portion of that weight is case, not boom.) Imagine if you had a traffic jam and something shorted one of them out.
                      $endgroup$
                      – Loren Pechtel
                      Dec 21 '18 at 21:47










                    • $begingroup$
                      Wouldn't the flywheels need to be used as contrarotating pairs to avoid changing the spin rate of the space station? (Space station Earth is large enough that they can be used individually.)
                      $endgroup$
                      – Andrew Morton
                      Dec 21 '18 at 21:58










                    • $begingroup$
                      Actually, flywheels work quite well as energy storage/transport, for instance the Gyrobus en.wikipedia.org/wiki/Gyrobus or the use of flywheels for energy storage in race cars popularmechanics.com/cars/hybrid-electric/a6266/… And of course every IC-engined car uses a flywheel as an energy buffer.
                      $endgroup$
                      – jamesqf
                      Dec 23 '18 at 20:31
















                    6












                    $begingroup$

                    Yes, but...



                    Flywheels are being used to store energy today, only not in any great amounts, and not as means of transporting energy from one place to another.



                    enter image description here



                    That big circle is a flywheel, acting as a sort of energy buffer between the strokes of the single cylinder of the locomotive.



                    In general, storing energy as momentum is a bad idea. You cannot store a lot of it before the flywheel is spinning so fast it tears itself apart, and unless you solve friction issues, you will quickly lose it as heat.



                    Also you get another problem: gyroscopic forces. Conservation of moment of inertia means that means you must mount these flywheels on some kind of gimbal, or you will never be able to turn your ship.



                    Finally, I must point out the inherent danger of having a huge rotating object inside your ship. Unlike chemical or nuclear fuel, you cannot just dump the energy and be rid of it. Also it means that your transport medium is extremely volatile. One thing that goes wrong with this... and you are in for a very bad time.






                    share|improve this answer









                    $endgroup$









                    • 1




                      $begingroup$
                      You are right that stored power in flywheels is dangerous, but large amounts stored power is always potentially dangerous if that power can ever be released in an uncontrolled manner.
                      $endgroup$
                      – Gary Walker
                      Dec 21 '18 at 20:01










                    • $begingroup$
                      Given the vacuum in space, I think you'd be better storing energy as heat instead of a rotating body.
                      $endgroup$
                      – Jiminion
                      Dec 21 '18 at 21:45










                    • $begingroup$
                      @GaryWalker Which is why we will never see a supercapacitor car. The energy is a typical automotive gas tank is similar to the energy in a normal 2000# military bomb. (Remember, a good portion of that weight is case, not boom.) Imagine if you had a traffic jam and something shorted one of them out.
                      $endgroup$
                      – Loren Pechtel
                      Dec 21 '18 at 21:47










                    • $begingroup$
                      Wouldn't the flywheels need to be used as contrarotating pairs to avoid changing the spin rate of the space station? (Space station Earth is large enough that they can be used individually.)
                      $endgroup$
                      – Andrew Morton
                      Dec 21 '18 at 21:58










                    • $begingroup$
                      Actually, flywheels work quite well as energy storage/transport, for instance the Gyrobus en.wikipedia.org/wiki/Gyrobus or the use of flywheels for energy storage in race cars popularmechanics.com/cars/hybrid-electric/a6266/… And of course every IC-engined car uses a flywheel as an energy buffer.
                      $endgroup$
                      – jamesqf
                      Dec 23 '18 at 20:31














                    6












                    6








                    6





                    $begingroup$

                    Yes, but...



                    Flywheels are being used to store energy today, only not in any great amounts, and not as means of transporting energy from one place to another.



                    enter image description here



                    That big circle is a flywheel, acting as a sort of energy buffer between the strokes of the single cylinder of the locomotive.



                    In general, storing energy as momentum is a bad idea. You cannot store a lot of it before the flywheel is spinning so fast it tears itself apart, and unless you solve friction issues, you will quickly lose it as heat.



                    Also you get another problem: gyroscopic forces. Conservation of moment of inertia means that means you must mount these flywheels on some kind of gimbal, or you will never be able to turn your ship.



                    Finally, I must point out the inherent danger of having a huge rotating object inside your ship. Unlike chemical or nuclear fuel, you cannot just dump the energy and be rid of it. Also it means that your transport medium is extremely volatile. One thing that goes wrong with this... and you are in for a very bad time.






                    share|improve this answer









                    $endgroup$



                    Yes, but...



                    Flywheels are being used to store energy today, only not in any great amounts, and not as means of transporting energy from one place to another.



                    enter image description here



                    That big circle is a flywheel, acting as a sort of energy buffer between the strokes of the single cylinder of the locomotive.



                    In general, storing energy as momentum is a bad idea. You cannot store a lot of it before the flywheel is spinning so fast it tears itself apart, and unless you solve friction issues, you will quickly lose it as heat.



                    Also you get another problem: gyroscopic forces. Conservation of moment of inertia means that means you must mount these flywheels on some kind of gimbal, or you will never be able to turn your ship.



                    Finally, I must point out the inherent danger of having a huge rotating object inside your ship. Unlike chemical or nuclear fuel, you cannot just dump the energy and be rid of it. Also it means that your transport medium is extremely volatile. One thing that goes wrong with this... and you are in for a very bad time.







                    share|improve this answer












                    share|improve this answer



                    share|improve this answer










                    answered Dec 21 '18 at 18:38









                    MichaelKMichaelK

                    36.6k691154




                    36.6k691154








                    • 1




                      $begingroup$
                      You are right that stored power in flywheels is dangerous, but large amounts stored power is always potentially dangerous if that power can ever be released in an uncontrolled manner.
                      $endgroup$
                      – Gary Walker
                      Dec 21 '18 at 20:01










                    • $begingroup$
                      Given the vacuum in space, I think you'd be better storing energy as heat instead of a rotating body.
                      $endgroup$
                      – Jiminion
                      Dec 21 '18 at 21:45










                    • $begingroup$
                      @GaryWalker Which is why we will never see a supercapacitor car. The energy is a typical automotive gas tank is similar to the energy in a normal 2000# military bomb. (Remember, a good portion of that weight is case, not boom.) Imagine if you had a traffic jam and something shorted one of them out.
                      $endgroup$
                      – Loren Pechtel
                      Dec 21 '18 at 21:47










                    • $begingroup$
                      Wouldn't the flywheels need to be used as contrarotating pairs to avoid changing the spin rate of the space station? (Space station Earth is large enough that they can be used individually.)
                      $endgroup$
                      – Andrew Morton
                      Dec 21 '18 at 21:58










                    • $begingroup$
                      Actually, flywheels work quite well as energy storage/transport, for instance the Gyrobus en.wikipedia.org/wiki/Gyrobus or the use of flywheels for energy storage in race cars popularmechanics.com/cars/hybrid-electric/a6266/… And of course every IC-engined car uses a flywheel as an energy buffer.
                      $endgroup$
                      – jamesqf
                      Dec 23 '18 at 20:31














                    • 1




                      $begingroup$
                      You are right that stored power in flywheels is dangerous, but large amounts stored power is always potentially dangerous if that power can ever be released in an uncontrolled manner.
                      $endgroup$
                      – Gary Walker
                      Dec 21 '18 at 20:01










                    • $begingroup$
                      Given the vacuum in space, I think you'd be better storing energy as heat instead of a rotating body.
                      $endgroup$
                      – Jiminion
                      Dec 21 '18 at 21:45










                    • $begingroup$
                      @GaryWalker Which is why we will never see a supercapacitor car. The energy is a typical automotive gas tank is similar to the energy in a normal 2000# military bomb. (Remember, a good portion of that weight is case, not boom.) Imagine if you had a traffic jam and something shorted one of them out.
                      $endgroup$
                      – Loren Pechtel
                      Dec 21 '18 at 21:47










                    • $begingroup$
                      Wouldn't the flywheels need to be used as contrarotating pairs to avoid changing the spin rate of the space station? (Space station Earth is large enough that they can be used individually.)
                      $endgroup$
                      – Andrew Morton
                      Dec 21 '18 at 21:58










                    • $begingroup$
                      Actually, flywheels work quite well as energy storage/transport, for instance the Gyrobus en.wikipedia.org/wiki/Gyrobus or the use of flywheels for energy storage in race cars popularmechanics.com/cars/hybrid-electric/a6266/… And of course every IC-engined car uses a flywheel as an energy buffer.
                      $endgroup$
                      – jamesqf
                      Dec 23 '18 at 20:31








                    1




                    1




                    $begingroup$
                    You are right that stored power in flywheels is dangerous, but large amounts stored power is always potentially dangerous if that power can ever be released in an uncontrolled manner.
                    $endgroup$
                    – Gary Walker
                    Dec 21 '18 at 20:01




                    $begingroup$
                    You are right that stored power in flywheels is dangerous, but large amounts stored power is always potentially dangerous if that power can ever be released in an uncontrolled manner.
                    $endgroup$
                    – Gary Walker
                    Dec 21 '18 at 20:01












                    $begingroup$
                    Given the vacuum in space, I think you'd be better storing energy as heat instead of a rotating body.
                    $endgroup$
                    – Jiminion
                    Dec 21 '18 at 21:45




                    $begingroup$
                    Given the vacuum in space, I think you'd be better storing energy as heat instead of a rotating body.
                    $endgroup$
                    – Jiminion
                    Dec 21 '18 at 21:45












                    $begingroup$
                    @GaryWalker Which is why we will never see a supercapacitor car. The energy is a typical automotive gas tank is similar to the energy in a normal 2000# military bomb. (Remember, a good portion of that weight is case, not boom.) Imagine if you had a traffic jam and something shorted one of them out.
                    $endgroup$
                    – Loren Pechtel
                    Dec 21 '18 at 21:47




                    $begingroup$
                    @GaryWalker Which is why we will never see a supercapacitor car. The energy is a typical automotive gas tank is similar to the energy in a normal 2000# military bomb. (Remember, a good portion of that weight is case, not boom.) Imagine if you had a traffic jam and something shorted one of them out.
                    $endgroup$
                    – Loren Pechtel
                    Dec 21 '18 at 21:47












                    $begingroup$
                    Wouldn't the flywheels need to be used as contrarotating pairs to avoid changing the spin rate of the space station? (Space station Earth is large enough that they can be used individually.)
                    $endgroup$
                    – Andrew Morton
                    Dec 21 '18 at 21:58




                    $begingroup$
                    Wouldn't the flywheels need to be used as contrarotating pairs to avoid changing the spin rate of the space station? (Space station Earth is large enough that they can be used individually.)
                    $endgroup$
                    – Andrew Morton
                    Dec 21 '18 at 21:58












                    $begingroup$
                    Actually, flywheels work quite well as energy storage/transport, for instance the Gyrobus en.wikipedia.org/wiki/Gyrobus or the use of flywheels for energy storage in race cars popularmechanics.com/cars/hybrid-electric/a6266/… And of course every IC-engined car uses a flywheel as an energy buffer.
                    $endgroup$
                    – jamesqf
                    Dec 23 '18 at 20:31




                    $begingroup$
                    Actually, flywheels work quite well as energy storage/transport, for instance the Gyrobus en.wikipedia.org/wiki/Gyrobus or the use of flywheels for energy storage in race cars popularmechanics.com/cars/hybrid-electric/a6266/… And of course every IC-engined car uses a flywheel as an energy buffer.
                    $endgroup$
                    – jamesqf
                    Dec 23 '18 at 20:31











                    6












                    $begingroup$

                    It is possible, but maybe not the best idea



                    Modern flywheels can accumulate a good amount of energy Energy densities, and they are relatively safe (meaning, they won't explosively disintegrate like solid flywheels can do), but they trail other means of energy storage in terms of energy density. Flywheel's specific energy, in MJ/kg, goes to 0.5, which is on par with regular Li-ion batteries, worse than Lithium metal batteries (1.8), and way, way worse than Hydrogen fuel cells (142). Actually, Wiki's figure for hydrogen assumes Earth conditions, where oxygen is plentiful. In spaceships, oxygen storage is also required. This would make the whole system several times less efficient per unit of mass, but still much better than a flywheel.



                    Flywheels are good when energy comes and goes in mechanical form. If you need the energy in a different form (to power the warp field) there will be extra overhead that would further negate their advantages.



                    But for creating a "streampunk universe", why not, go with flywheels!






                    share|improve this answer











                    $endgroup$


















                      6












                      $begingroup$

                      It is possible, but maybe not the best idea



                      Modern flywheels can accumulate a good amount of energy Energy densities, and they are relatively safe (meaning, they won't explosively disintegrate like solid flywheels can do), but they trail other means of energy storage in terms of energy density. Flywheel's specific energy, in MJ/kg, goes to 0.5, which is on par with regular Li-ion batteries, worse than Lithium metal batteries (1.8), and way, way worse than Hydrogen fuel cells (142). Actually, Wiki's figure for hydrogen assumes Earth conditions, where oxygen is plentiful. In spaceships, oxygen storage is also required. This would make the whole system several times less efficient per unit of mass, but still much better than a flywheel.



                      Flywheels are good when energy comes and goes in mechanical form. If you need the energy in a different form (to power the warp field) there will be extra overhead that would further negate their advantages.



                      But for creating a "streampunk universe", why not, go with flywheels!






                      share|improve this answer











                      $endgroup$
















                        6












                        6








                        6





                        $begingroup$

                        It is possible, but maybe not the best idea



                        Modern flywheels can accumulate a good amount of energy Energy densities, and they are relatively safe (meaning, they won't explosively disintegrate like solid flywheels can do), but they trail other means of energy storage in terms of energy density. Flywheel's specific energy, in MJ/kg, goes to 0.5, which is on par with regular Li-ion batteries, worse than Lithium metal batteries (1.8), and way, way worse than Hydrogen fuel cells (142). Actually, Wiki's figure for hydrogen assumes Earth conditions, where oxygen is plentiful. In spaceships, oxygen storage is also required. This would make the whole system several times less efficient per unit of mass, but still much better than a flywheel.



                        Flywheels are good when energy comes and goes in mechanical form. If you need the energy in a different form (to power the warp field) there will be extra overhead that would further negate their advantages.



                        But for creating a "streampunk universe", why not, go with flywheels!






                        share|improve this answer











                        $endgroup$



                        It is possible, but maybe not the best idea



                        Modern flywheels can accumulate a good amount of energy Energy densities, and they are relatively safe (meaning, they won't explosively disintegrate like solid flywheels can do), but they trail other means of energy storage in terms of energy density. Flywheel's specific energy, in MJ/kg, goes to 0.5, which is on par with regular Li-ion batteries, worse than Lithium metal batteries (1.8), and way, way worse than Hydrogen fuel cells (142). Actually, Wiki's figure for hydrogen assumes Earth conditions, where oxygen is plentiful. In spaceships, oxygen storage is also required. This would make the whole system several times less efficient per unit of mass, but still much better than a flywheel.



                        Flywheels are good when energy comes and goes in mechanical form. If you need the energy in a different form (to power the warp field) there will be extra overhead that would further negate their advantages.



                        But for creating a "streampunk universe", why not, go with flywheels!







                        share|improve this answer














                        share|improve this answer



                        share|improve this answer








                        edited Dec 21 '18 at 19:51

























                        answered Dec 21 '18 at 19:08









                        AlexanderAlexander

                        19.7k53175




                        19.7k53175























                            4












                            $begingroup$

                            Flywheels are not ideal for your application in the 25th century.



                            Due to materials advances, flywheels would have a larger capacity than today, but there are 2 reasonable tech advances that would likely be much nicer.



                            Super-capacitors have already been proposed that have energy densities equal to fuel cells. They also have power densities that are orders of magnitudes better. It is hard to predict just how powerful these could become with an additional 400 years of advances.



                            Room temperature superconductors may lead to high capacity storage as well.



                            Both of these technologies are inherently electrical instead of mechanical, and potentially more usable for directly powering a warp field.



                            You specify powering a warp gate for hours or more, these technologies would be suitable for this, or for powering warp gates for minutes or even seconds.



                            Flywheels are rarely used as a power source for hours, they are more commonly used as a power source for minutes or seconds until a secondary power source can respond (such as a diesel generator). This is because the storage densities of flywheels is relatively low.



                            But spinning metal is very good at averaging power fluctuations. In fact, spinning metal in the generators themselves is a large component in power averaging the current electric grid.



                            If you want another solution, consider the possibility of using hafnium, specifically 178m2Hf. You need technical advances to figure out how to release the stored energy when desired, but the storage potential is huge. This adds complication in radio-activity and how to convert the gamma energy-release into something for powering your warp-generators, but this could be woven into an interesting plot-line.






                            share|improve this answer











                            $endgroup$


















                              4












                              $begingroup$

                              Flywheels are not ideal for your application in the 25th century.



                              Due to materials advances, flywheels would have a larger capacity than today, but there are 2 reasonable tech advances that would likely be much nicer.



                              Super-capacitors have already been proposed that have energy densities equal to fuel cells. They also have power densities that are orders of magnitudes better. It is hard to predict just how powerful these could become with an additional 400 years of advances.



                              Room temperature superconductors may lead to high capacity storage as well.



                              Both of these technologies are inherently electrical instead of mechanical, and potentially more usable for directly powering a warp field.



                              You specify powering a warp gate for hours or more, these technologies would be suitable for this, or for powering warp gates for minutes or even seconds.



                              Flywheels are rarely used as a power source for hours, they are more commonly used as a power source for minutes or seconds until a secondary power source can respond (such as a diesel generator). This is because the storage densities of flywheels is relatively low.



                              But spinning metal is very good at averaging power fluctuations. In fact, spinning metal in the generators themselves is a large component in power averaging the current electric grid.



                              If you want another solution, consider the possibility of using hafnium, specifically 178m2Hf. You need technical advances to figure out how to release the stored energy when desired, but the storage potential is huge. This adds complication in radio-activity and how to convert the gamma energy-release into something for powering your warp-generators, but this could be woven into an interesting plot-line.






                              share|improve this answer











                              $endgroup$
















                                4












                                4








                                4





                                $begingroup$

                                Flywheels are not ideal for your application in the 25th century.



                                Due to materials advances, flywheels would have a larger capacity than today, but there are 2 reasonable tech advances that would likely be much nicer.



                                Super-capacitors have already been proposed that have energy densities equal to fuel cells. They also have power densities that are orders of magnitudes better. It is hard to predict just how powerful these could become with an additional 400 years of advances.



                                Room temperature superconductors may lead to high capacity storage as well.



                                Both of these technologies are inherently electrical instead of mechanical, and potentially more usable for directly powering a warp field.



                                You specify powering a warp gate for hours or more, these technologies would be suitable for this, or for powering warp gates for minutes or even seconds.



                                Flywheels are rarely used as a power source for hours, they are more commonly used as a power source for minutes or seconds until a secondary power source can respond (such as a diesel generator). This is because the storage densities of flywheels is relatively low.



                                But spinning metal is very good at averaging power fluctuations. In fact, spinning metal in the generators themselves is a large component in power averaging the current electric grid.



                                If you want another solution, consider the possibility of using hafnium, specifically 178m2Hf. You need technical advances to figure out how to release the stored energy when desired, but the storage potential is huge. This adds complication in radio-activity and how to convert the gamma energy-release into something for powering your warp-generators, but this could be woven into an interesting plot-line.






                                share|improve this answer











                                $endgroup$



                                Flywheels are not ideal for your application in the 25th century.



                                Due to materials advances, flywheels would have a larger capacity than today, but there are 2 reasonable tech advances that would likely be much nicer.



                                Super-capacitors have already been proposed that have energy densities equal to fuel cells. They also have power densities that are orders of magnitudes better. It is hard to predict just how powerful these could become with an additional 400 years of advances.



                                Room temperature superconductors may lead to high capacity storage as well.



                                Both of these technologies are inherently electrical instead of mechanical, and potentially more usable for directly powering a warp field.



                                You specify powering a warp gate for hours or more, these technologies would be suitable for this, or for powering warp gates for minutes or even seconds.



                                Flywheels are rarely used as a power source for hours, they are more commonly used as a power source for minutes or seconds until a secondary power source can respond (such as a diesel generator). This is because the storage densities of flywheels is relatively low.



                                But spinning metal is very good at averaging power fluctuations. In fact, spinning metal in the generators themselves is a large component in power averaging the current electric grid.



                                If you want another solution, consider the possibility of using hafnium, specifically 178m2Hf. You need technical advances to figure out how to release the stored energy when desired, but the storage potential is huge. This adds complication in radio-activity and how to convert the gamma energy-release into something for powering your warp-generators, but this could be woven into an interesting plot-line.







                                share|improve this answer














                                share|improve this answer



                                share|improve this answer








                                edited Dec 21 '18 at 20:09

























                                answered Dec 21 '18 at 19:51









                                Gary WalkerGary Walker

                                14.8k22754




                                14.8k22754























                                    3












                                    $begingroup$

                                    Your very awesome flywheel is made of neutronium.



                                    binary neutron stars



                                    https://newscenter.lbl.gov/2017/10/16/scientists-decode-signature-neutron-star-merger/



                                    A flywheel will tear itself apart if it rotates too fast. Unless the forces holding the flywheel together are super strong - as is the case for neutron stars. Neutron stars will teach you about fast spinning.




                                    As the star's core collapses, its rotation rate increases as a result
                                    of conservation of angular momentum, hence newly formed neutron stars
                                    rotate at up to several hundred times per second.




                                    The neutronium comprising neutron stars is so dense that it will stay together even at relativistic rotation speeds. A 10 km neutron star with the mass of the Sun might be a little unwieldy for your application. I wondered if a smaller chunk of neutronium could still hold together. According to the physics stack, maybe.



                                    https://physics.stackexchange.com/questions/143166/what-is-the-theoretical-lower-mass-limit-for-a-gravitationally-stable-neutron-st




                                    Theoretically a stable neutron star could exist with a much lower
                                    mass, if one could work out a way of forming it (perhaps in a close
                                    binary neutron star where one component loses mass to the other prior
                                    to a merger?).




                                    It goes on with some hard core physics, including why a smaller amount of neutronium might make a larger object that degenerates into more commonplace matter.



                                    Or maybe you would site your stargates at naturally occurring binary neutron stars, as depicted above. In any case - your flywheel is a spinning chunk of neutronium. Or two spinning chunks, to form the halves of your gate as noted in the OP. Given their mass they will be orbiting around each other. Given their proximity they will be moving very, very fast and they will look very very cool. You can approach perpendicular to their plane of motion and pass between them. You had better stay in the exact center. Stars get grabby.



                                    How does one get energy from a fast rotating neutron star? Maybe induce it to shoot out a jet of energy that you harness for your purposes.



                                    https://www.sciencedaily.com/releases/2018/09/180926140829.htm




                                    Another idea, they said, is that the jets may be powered by the
                                    neutron star's rotation, instead of being launched by magnetic field
                                    lines in the inner accretion disk.



                                    "Interestingly, the rotation-powered idea predicts that the jet will
                                    be significantly weaker from more slowly rotating neutron stars, which
                                    is exactly what we see in Sw J0243," Nathalie Degenaar, also of the
                                    University of Amsterdam, said.







                                    share|improve this answer











                                    $endgroup$


















                                      3












                                      $begingroup$

                                      Your very awesome flywheel is made of neutronium.



                                      binary neutron stars



                                      https://newscenter.lbl.gov/2017/10/16/scientists-decode-signature-neutron-star-merger/



                                      A flywheel will tear itself apart if it rotates too fast. Unless the forces holding the flywheel together are super strong - as is the case for neutron stars. Neutron stars will teach you about fast spinning.




                                      As the star's core collapses, its rotation rate increases as a result
                                      of conservation of angular momentum, hence newly formed neutron stars
                                      rotate at up to several hundred times per second.




                                      The neutronium comprising neutron stars is so dense that it will stay together even at relativistic rotation speeds. A 10 km neutron star with the mass of the Sun might be a little unwieldy for your application. I wondered if a smaller chunk of neutronium could still hold together. According to the physics stack, maybe.



                                      https://physics.stackexchange.com/questions/143166/what-is-the-theoretical-lower-mass-limit-for-a-gravitationally-stable-neutron-st




                                      Theoretically a stable neutron star could exist with a much lower
                                      mass, if one could work out a way of forming it (perhaps in a close
                                      binary neutron star where one component loses mass to the other prior
                                      to a merger?).




                                      It goes on with some hard core physics, including why a smaller amount of neutronium might make a larger object that degenerates into more commonplace matter.



                                      Or maybe you would site your stargates at naturally occurring binary neutron stars, as depicted above. In any case - your flywheel is a spinning chunk of neutronium. Or two spinning chunks, to form the halves of your gate as noted in the OP. Given their mass they will be orbiting around each other. Given their proximity they will be moving very, very fast and they will look very very cool. You can approach perpendicular to their plane of motion and pass between them. You had better stay in the exact center. Stars get grabby.



                                      How does one get energy from a fast rotating neutron star? Maybe induce it to shoot out a jet of energy that you harness for your purposes.



                                      https://www.sciencedaily.com/releases/2018/09/180926140829.htm




                                      Another idea, they said, is that the jets may be powered by the
                                      neutron star's rotation, instead of being launched by magnetic field
                                      lines in the inner accretion disk.



                                      "Interestingly, the rotation-powered idea predicts that the jet will
                                      be significantly weaker from more slowly rotating neutron stars, which
                                      is exactly what we see in Sw J0243," Nathalie Degenaar, also of the
                                      University of Amsterdam, said.







                                      share|improve this answer











                                      $endgroup$
















                                        3












                                        3








                                        3





                                        $begingroup$

                                        Your very awesome flywheel is made of neutronium.



                                        binary neutron stars



                                        https://newscenter.lbl.gov/2017/10/16/scientists-decode-signature-neutron-star-merger/



                                        A flywheel will tear itself apart if it rotates too fast. Unless the forces holding the flywheel together are super strong - as is the case for neutron stars. Neutron stars will teach you about fast spinning.




                                        As the star's core collapses, its rotation rate increases as a result
                                        of conservation of angular momentum, hence newly formed neutron stars
                                        rotate at up to several hundred times per second.




                                        The neutronium comprising neutron stars is so dense that it will stay together even at relativistic rotation speeds. A 10 km neutron star with the mass of the Sun might be a little unwieldy for your application. I wondered if a smaller chunk of neutronium could still hold together. According to the physics stack, maybe.



                                        https://physics.stackexchange.com/questions/143166/what-is-the-theoretical-lower-mass-limit-for-a-gravitationally-stable-neutron-st




                                        Theoretically a stable neutron star could exist with a much lower
                                        mass, if one could work out a way of forming it (perhaps in a close
                                        binary neutron star where one component loses mass to the other prior
                                        to a merger?).




                                        It goes on with some hard core physics, including why a smaller amount of neutronium might make a larger object that degenerates into more commonplace matter.



                                        Or maybe you would site your stargates at naturally occurring binary neutron stars, as depicted above. In any case - your flywheel is a spinning chunk of neutronium. Or two spinning chunks, to form the halves of your gate as noted in the OP. Given their mass they will be orbiting around each other. Given their proximity they will be moving very, very fast and they will look very very cool. You can approach perpendicular to their plane of motion and pass between them. You had better stay in the exact center. Stars get grabby.



                                        How does one get energy from a fast rotating neutron star? Maybe induce it to shoot out a jet of energy that you harness for your purposes.



                                        https://www.sciencedaily.com/releases/2018/09/180926140829.htm




                                        Another idea, they said, is that the jets may be powered by the
                                        neutron star's rotation, instead of being launched by magnetic field
                                        lines in the inner accretion disk.



                                        "Interestingly, the rotation-powered idea predicts that the jet will
                                        be significantly weaker from more slowly rotating neutron stars, which
                                        is exactly what we see in Sw J0243," Nathalie Degenaar, also of the
                                        University of Amsterdam, said.







                                        share|improve this answer











                                        $endgroup$



                                        Your very awesome flywheel is made of neutronium.



                                        binary neutron stars



                                        https://newscenter.lbl.gov/2017/10/16/scientists-decode-signature-neutron-star-merger/



                                        A flywheel will tear itself apart if it rotates too fast. Unless the forces holding the flywheel together are super strong - as is the case for neutron stars. Neutron stars will teach you about fast spinning.




                                        As the star's core collapses, its rotation rate increases as a result
                                        of conservation of angular momentum, hence newly formed neutron stars
                                        rotate at up to several hundred times per second.




                                        The neutronium comprising neutron stars is so dense that it will stay together even at relativistic rotation speeds. A 10 km neutron star with the mass of the Sun might be a little unwieldy for your application. I wondered if a smaller chunk of neutronium could still hold together. According to the physics stack, maybe.



                                        https://physics.stackexchange.com/questions/143166/what-is-the-theoretical-lower-mass-limit-for-a-gravitationally-stable-neutron-st




                                        Theoretically a stable neutron star could exist with a much lower
                                        mass, if one could work out a way of forming it (perhaps in a close
                                        binary neutron star where one component loses mass to the other prior
                                        to a merger?).




                                        It goes on with some hard core physics, including why a smaller amount of neutronium might make a larger object that degenerates into more commonplace matter.



                                        Or maybe you would site your stargates at naturally occurring binary neutron stars, as depicted above. In any case - your flywheel is a spinning chunk of neutronium. Or two spinning chunks, to form the halves of your gate as noted in the OP. Given their mass they will be orbiting around each other. Given their proximity they will be moving very, very fast and they will look very very cool. You can approach perpendicular to their plane of motion and pass between them. You had better stay in the exact center. Stars get grabby.



                                        How does one get energy from a fast rotating neutron star? Maybe induce it to shoot out a jet of energy that you harness for your purposes.



                                        https://www.sciencedaily.com/releases/2018/09/180926140829.htm




                                        Another idea, they said, is that the jets may be powered by the
                                        neutron star's rotation, instead of being launched by magnetic field
                                        lines in the inner accretion disk.



                                        "Interestingly, the rotation-powered idea predicts that the jet will
                                        be significantly weaker from more slowly rotating neutron stars, which
                                        is exactly what we see in Sw J0243," Nathalie Degenaar, also of the
                                        University of Amsterdam, said.








                                        share|improve this answer














                                        share|improve this answer



                                        share|improve this answer








                                        edited Dec 22 '18 at 21:57

























                                        answered Dec 22 '18 at 21:49









                                        WillkWillk

                                        104k25197440




                                        104k25197440























                                            0












                                            $begingroup$

                                            If you need all your energy to be released in one go (is in the case of generator stations) you have significantly fewer options than normal. Any thermal method of energy generation will likely be too hot for most materials and batteries are simply too slow to release your energy. This leaves you with just capacitors, flywheels and maybe handwavium if you've got it. Though in all honesty this explanation requires the bulk your energy to be released very quickly for batteries to be non-competitive.



                                            (in space) Flywheels are very very cheap. This is because they are incredibly simple. if you find a significantly massive asteroid (of which there will probably be millions in any given solar system) you can simply smelt it into steel (quite cheap given the awsome source of energy that is fusion) and then BAMN flywheel, much more processing would be required for a given energy of capacitor bank.) The flywheel in question would be massive though.



                                            With good barings your flywheel will basically lose no energy to the outside would



                                            Lets say you have a 100 MW reactor given a energy density of 32 Watt-hours per kilogram storing energy over a week then that's going to be 500000 tons of steel. That's about 70 effiel towers. This is definitely doable just make sure to have your flywheels by BIG.






                                            share|improve this answer









                                            $endgroup$


















                                              0












                                              $begingroup$

                                              If you need all your energy to be released in one go (is in the case of generator stations) you have significantly fewer options than normal. Any thermal method of energy generation will likely be too hot for most materials and batteries are simply too slow to release your energy. This leaves you with just capacitors, flywheels and maybe handwavium if you've got it. Though in all honesty this explanation requires the bulk your energy to be released very quickly for batteries to be non-competitive.



                                              (in space) Flywheels are very very cheap. This is because they are incredibly simple. if you find a significantly massive asteroid (of which there will probably be millions in any given solar system) you can simply smelt it into steel (quite cheap given the awsome source of energy that is fusion) and then BAMN flywheel, much more processing would be required for a given energy of capacitor bank.) The flywheel in question would be massive though.



                                              With good barings your flywheel will basically lose no energy to the outside would



                                              Lets say you have a 100 MW reactor given a energy density of 32 Watt-hours per kilogram storing energy over a week then that's going to be 500000 tons of steel. That's about 70 effiel towers. This is definitely doable just make sure to have your flywheels by BIG.






                                              share|improve this answer









                                              $endgroup$
















                                                0












                                                0








                                                0





                                                $begingroup$

                                                If you need all your energy to be released in one go (is in the case of generator stations) you have significantly fewer options than normal. Any thermal method of energy generation will likely be too hot for most materials and batteries are simply too slow to release your energy. This leaves you with just capacitors, flywheels and maybe handwavium if you've got it. Though in all honesty this explanation requires the bulk your energy to be released very quickly for batteries to be non-competitive.



                                                (in space) Flywheels are very very cheap. This is because they are incredibly simple. if you find a significantly massive asteroid (of which there will probably be millions in any given solar system) you can simply smelt it into steel (quite cheap given the awsome source of energy that is fusion) and then BAMN flywheel, much more processing would be required for a given energy of capacitor bank.) The flywheel in question would be massive though.



                                                With good barings your flywheel will basically lose no energy to the outside would



                                                Lets say you have a 100 MW reactor given a energy density of 32 Watt-hours per kilogram storing energy over a week then that's going to be 500000 tons of steel. That's about 70 effiel towers. This is definitely doable just make sure to have your flywheels by BIG.






                                                share|improve this answer









                                                $endgroup$



                                                If you need all your energy to be released in one go (is in the case of generator stations) you have significantly fewer options than normal. Any thermal method of energy generation will likely be too hot for most materials and batteries are simply too slow to release your energy. This leaves you with just capacitors, flywheels and maybe handwavium if you've got it. Though in all honesty this explanation requires the bulk your energy to be released very quickly for batteries to be non-competitive.



                                                (in space) Flywheels are very very cheap. This is because they are incredibly simple. if you find a significantly massive asteroid (of which there will probably be millions in any given solar system) you can simply smelt it into steel (quite cheap given the awsome source of energy that is fusion) and then BAMN flywheel, much more processing would be required for a given energy of capacitor bank.) The flywheel in question would be massive though.



                                                With good barings your flywheel will basically lose no energy to the outside would



                                                Lets say you have a 100 MW reactor given a energy density of 32 Watt-hours per kilogram storing energy over a week then that's going to be 500000 tons of steel. That's about 70 effiel towers. This is definitely doable just make sure to have your flywheels by BIG.







                                                share|improve this answer












                                                share|improve this answer



                                                share|improve this answer










                                                answered Dec 21 '18 at 19:54









                                                UmmdustryUmmdustry

                                                3,734822




                                                3,734822






























                                                    draft saved

                                                    draft discarded




















































                                                    Thanks for contributing an answer to Worldbuilding Stack Exchange!


                                                    • Please be sure to answer the question. Provide details and share your research!

                                                    But avoid



                                                    • Asking for help, clarification, or responding to other answers.

                                                    • Making statements based on opinion; back them up with references or personal experience.


                                                    Use MathJax to format equations. MathJax reference.


                                                    To learn more, see our tips on writing great answers.




                                                    draft saved


                                                    draft discarded














                                                    StackExchange.ready(
                                                    function () {
                                                    StackExchange.openid.initPostLogin('.new-post-login', 'https%3a%2f%2fworldbuilding.stackexchange.com%2fquestions%2f134515%2fare-fly-wheels-a-valid-means-of-energy-storage-for-powering-a-large-space-statio%23new-answer', 'question_page');
                                                    }
                                                    );

                                                    Post as a guest















                                                    Required, but never shown





















































                                                    Required, but never shown














                                                    Required, but never shown












                                                    Required, but never shown







                                                    Required, but never shown

































                                                    Required, but never shown














                                                    Required, but never shown












                                                    Required, but never shown







                                                    Required, but never shown







                                                    Popular posts from this blog

                                                    Plaza Victoria

                                                    In PowerPoint, is there a keyboard shortcut for bulleted / numbered list?

                                                    How to put 3 figures in Latex with 2 figures side by side and 1 below these side by side images but in...